PhysicsPhysics QuestionsMoving Charges And Magnetism Questions for CBSE Class 12th

Moving Charges And Magnetism Questions for CBSE Class 12th

A particle of mass 2 x 10 5 kg moves horizontally between two horizontal plates of a charged parallel plate capacitor between which there is an electric field of 200 NC -1 acting upward. A magnetic induction of 2.0 T is applied at right angles to the electric field in a direction normal to both B and v . If g is 9.8 m s -2 and the charge on the particle is 10 -6 C, then find the velocity of charge particle so that it continues to move horizontally

A charged particle of mass 10 -3 kg and charge 10 -5 C enters a magnetic field of induction 1T. If g= 10 ms -2 , for what value of velocity will it pass straight through the field without deflection?

    Fill Out the Form for Expert Academic Guidance!



    +91


    Live ClassesBooksTest SeriesSelf Learning




    Verify OTP Code (required)

    I agree to the terms and conditions and privacy policy.

    An electron is moving along positive x-axis. To get it moving on an anticlockwise circular path in x-y plane, a magnetic field is applied

    Three particles, an electron (e), a proton (p) and a helium atom (He) are moving in circular paths with constant speed in the x-y plane in a region where a uniform magnetic field B exists along z-axis. The times taken by e, p and He inside the field to complete one revolution are t e , t p and t He respectively.Then

    A particle of specific charge q m = π Ckg – 1 is projected from the origin toward positive x-axis with a velocity of 10 m s – 1 in a uniform magnetic field B = – 2 k ^ T . The velocity v of particle after time t= 1/12 s will be (in ms -1 )

    A semicircular wire of radius R, carrying current i, is placed in a magnetic field as shown in figure. On left side of X’X, magnetic field strength is B 0 , and on right side of X’X,magnetic field strength is 2B 0 . Both fields are directed inside the page.The magnetic force experienced by the wire would be

    Consider a hypothetic spherical body.The body is cut into two parts about the diameter. One of hemispherical portion has mass distribution m while the other portion has identical charge distribution q. The body is rotated about the axis with constant speed ω . Then, the ratio of magnetic moment to angular momentum is

    The magnetic induction at center O (Figure) is

    Two circular coils of radii 5 cm and 10 cm carry equal currents of 2 A. The coils have 50 and 100 turns, respectively and are placed in such a way that their planes as well as their centers coincide. Magnitude of magnetic field at the common center of coils is

    A charged particle moves with velocity v = a i ^ + d j ^ in a magnetic field B = A i ^ + D j ^ . The force acting on the particle has magnitude F. Then,

    A proton and an α -particle enter a uniform magnetic field moving with the same speed. If the proton takes 25 μ s to make 5 revolutions, then the periodic time for the α -particle would be

    A circular coil having mass m is kept above ground (x-z plane) at some height. The coil carries a current i in the direction shown in figure. In which direction a uniform magnetic field B be applied so that the magnetic force balances the weight of the coil?

    An electron is projected at an angle θ with a uniform magnetic field. If the pitch of the helical path is equal to its radius, then the angle of projection is

    There exist uniform magnetic and electric fields of magnitudes 1 T and 1 V m -1 , respectively, along positive y-axis. A charged particle of mass 1 kg and charge 1 C is having velocity 1 ms -1 along x-axis and is at origin at t= 0. Then, the coordinates of the particle at time π s will be

    A current carrying loop is placed in the non-uniform magnetic field whose variation in space is shown in figure.Direction of magnetic field is into the plane of paper. The magnetic force experienced by the loop is

    Let current i = 2 A be flowing in each part of a wire frame as shown in figure. The frame is a combination of two equilateral triangles ACD and CDE of side 1 m. It is placed in uniform magnetic field B = 4T acting perpendicular to the plane of frame.The magnitude of magnetic force acting on the frame is

    A uniform current carrying ring of mass m and radius R is connected by a mass less string as shown in figure. A uniform magnetic field B 0 , exists in the region to keep the ring in horizontal position, then the current in the ring is (l = length of string)

    A wire of cross-sectional area A forms three sides of a square and is free to rotate about axis OO 1 . If the structure is deflected by an angle θ from the vertical when current I is passed through it in a magnetic field B acting vertically upward and density of the wire is ρ , then the value of θ is given by

    A straight piece of conducting wire with mass M and length L is placed on a friction less incline tilted at an angle θ from the horizontal (as shown in figure). There is a uniform, vertical magnetic field at all points (produced by an arrangement of magnets not shown in figure). To keep the wire from sliding down the incline, a voltage source is attached to the ends of the wire. When just the right amount of current flows through the wire, the wire remains at rest.Determine the magnitude and direction of the current in the wire that will cause the wire to remain at rest.

    Find the magnitude of the magnetic induction B of a magnetic field generated by a system of thin conductors along which a current i is flowing at a point A (O, R, O), that is the centre of a circular conductor of radius R. The ring is in yz plane.

    A parabolic section of wire OA is located in the x-y plane and carries current I = 12 A. A uniform magnetic field B=4.0T making an angle 60 0 with x axis exists in x-y plane.Calculate the magnetic force on the wire OA. Co-ordinates of A are (0.25 m, 1 m).

    A triangular system of mass 100 g consisting of 3 wires of lengths, as shown in figure and length of AO as 4 units, are placed in the magnetic field of 1T. The current of 1A flows through wire AO. The wires are of same material and cross-sectional area. In which direction will the system move?

    A proton beam passes without deviation through a region of space where there are uniform transverse mutually perpendicular electric and magnetic field with E = 120 kV m and B = 50 mT . Then, the beam strikes a grounded target. Find the force imparted by the beam on the target (in μ N) if the beam current is equal to I = 0 .80 mA . Take mass of proton as 1 . 67 × 10 – 27 kg .

    A particle of specific charge q / m = π C / kg is projected from the origin towards positive x-axis with a velocity of 10 m/s in a uniform magnetic field B = − 2 k ^ Tesla. The velocity V of the particle after time t = 1/6 s will be

    A charged particle (electron or proton) is introduced at the origin (x=0,y=0,z=0) with a given initial velocity ν . A uniform electric field E and a uniform magnetic field B exist everywhere. The velocity ν , electric field E and magnetic field B are given in columns 1,2 and 3, respectively. The quantities E 0 , B 0 are positive in magnitude. Column 1 Column 2 Column 3 (I) Electron with ν = 2 E 0 B 0 x ^ (i) E = E 0 z ^ (P) B = − B 0 x ^ (II) Electron with ν = E 0 B 0 y ^ (ii) E = − E 0 y ^ (Q) B = B 0 x ^ (III) Proton with ν = 0 (iii) E = − E 0 x ^ (R) B = B 0 y ^ (IV) Proton with ν = 2 E 0 B 0 x ^ (iv) E = E 0 x ^ (S) B = B 0 z ^ In which case would the particle move in a straight line along the negative direction of y-axis (i.e., move along – y ^ )?

    An infinitely long straight wire is carrying a current I 1 . Adjacent to it there is another equilateral triangular wire having current I 2 . Choose the wrong options.

    Two particles Y and Z emitted by a radioactive source at P made tracks in a chamber as illustrated in the figure.A magnetic field acts downward into the paper. Careful measurements showed that both tracks were circular, the radius of Y track being half that of the Z track. Which one of the following statements is certainly true?

    An electron and a proton each travel with equal speeds around circular orbits in the same uniform magnetic field as indicated (not to scale) in figure. The field is into the page on the diagram. The electron travels …….. around the ……..circle and the proton travels ……..around the……..circle.

    In a region of space, a uniform magnetic field B exists in the x-direction. An electron is fired from the origin with its initial velocity making an angle with they direction in the y-z plane. In the subsequent motion of the electron,

    A charged particle is whirled in a horizontal circle on a frictionless table by attaching it to a string fixed at one end.If a magnetic field is switched on in the vertical direction,the tension in the string

    A charged particle moves along a circle under the action of possible constant electric and magnetic fields. Which of the following are possible?

    An electron is launched with velocity v in a uniform magnetic field B . The angle θ between V and B lies between 0 and π 2 . Its velocity vector v returns to its initial value in a time interval of

    A charged particle begins to move from the origin in a region which has a uniform magnetic field in the x-direction and a uniform electric field in the y-direction. Its speed is v when it reaches the point (x, y, z). Then, v will NOT depend

    A square loop of wire carrying current i is lying in the plane of paper as shown in figure. The magnetic field is present in the region as shown. The loop will tend to rotate

    The rigid conducting thin wire frame carries an electric current I and this frame is inside a uniform magnetic field B as shown in figure. Then,

    A charged particle moves in a uniform magnetic field perpendicular to it, in a circle of radius of 4 cm. On passing through a metallic sheet it loses half of its kinetic energy. Then, the radius of curvature of the particle is

    A uniform magnetic field exists in a region which forms an equilateral triangle of side a. The magnetic field is perpendicular to the plane of the triangle. A charge q enters into this magnetic field perpendicular to a side with speed v.The charge enters from midpoint and leaves the field from mid-point of other side. Magnetic induction in the triangle is

    A conducting rod of mass m and length l is placed over a smooth horizontal surface. A uniform magnetic field B is acting perpendicular to the rod. Charge q is suddenly passed through the rod and it acquires an initial velocity v on the surface, then q is equal to

    The plane of a rectangular loop of wire with sides 0.05 m and 0.08 m is parallel to a uniform magnetic field of induction 1.5 x 10 -2 T. A current of 10.0 ampere flows through the loop. If the side of length 0.08 m is normal and the side of length 0.05 m is parallel to the lines of induction, then the torque acting on the loop is

    An electron is accelerated from rest through a potential difference V. This electron experiences a force F in a uniform magnetic field. On increasing the potential difference to V’,the force experienced by the electron in the same magnetic field becomes 2F. Then, the ratio (V’/V) is equal to

    Figure shows a conducting loop ABCDA placed in a uniform magnetic field (strength B) perpendicular to its plane. The part ABC is the (three-fourth) portion of the square of side length l. The part ADC is a circular arc of radius R. The points A and C are connected to a, battery which supplies a current i to the circuit.The magnetic force on the loop due to the field B is

    A uniform conducting rectangular loop of sides l, b and mass m carrying current i is hanging horizontally with the help of two vertical strings. There exists a uniform horizontal magnetic field B which is parallel to the longer side of loop.The value of tension which is least is

    A conducting rod of length / and mass m is moving down a smooth inclined plane of inclination θ with constant velocity v. A current i is flowing in the conductor in a direction perpendicular to paper inwards. A vertically upward magnetic field B exists in space. Then, magnitude of magnetic field B is

    A uniform field B = 3 i ^ + 4 j ^ + k ^ exists in region of space. A semicircular wire of radius 1 m carrying current 1A having its centre at (2, 2, 0) is placed in x-y plane as shown in figure. The force on semi circular wire will be

    An electron gun ejects electrons at an angle of 45 0 with magnetic field boundary as shown in figure. Find the angular deviation of electrons as it comes out of field.

    An α -particle and a proton are both simultaneously projected in opposite directions into a region of constant magnetic field perpendicular to the direction of the field.After some time it is found that the velocity of the a-particle has changed in a direction by 45 0 . Then at this time, the angle between velocity vectors of α -particle and proton is

    A uniform magnetic field B and electric field E exist along y and negative z axis respectively. Under the influence of these fields a charge particle moves along OA undeflected. If electric field is switched off, find the pitch of helical trajectory in which the particle will move.

    AB and AC are the boundary lines within which a magnetic field B exists. If the magnetic field is absent, a charged particle of mass m and charge q must have passed through a point P on angle bisector of —BAC, at a distance r 2 from A, if it has fallen on AB normally at a point Q such that OP = r. If the magnetic field is present, how much time the charged particle will take to come out of the magnetic field

    To the right of line PQ is a uniform magnetic field B. B 1 A is the line of incidence of a charged particle, which comes out of the field along DE; CA and DF are the normals at A and D. B 1 AC = θ . Angle measured from CA in clockwise direction is taken as positive. The maximum range of movement of the centre of the part of the circle from line AD in which charged particle of charge Q moves with a velocity v when θ is positive to when θ is negative is given by

    Current I flows through the circuit, as shown in figure. Find the magnetic moment of the figure, if AB = BC = CD = DE= EF = FG=GH=HA.

    A triangular system of mass 100 g consisting of 3 wires of lengths, as shown in figure and length of AO as 4 units, are placed in the magnetic field of 1T. The current of 1A flows through wire AO. The wires are of same material and cross-sectional area. what will be acceleration of the system (in m/sec 2 ) ?

    A straight wire carrying current is parallel to the y-axis as shown in figure. The

    A particle of charge q and mass m starts moving from the origin under the action of an electric field E = E 0 i ^ and B = B 0 i ^ with a velocity v = v 0 j ^ . The speed of the particle will become 2v 0 after a time

    A charged particle P leaves the origin with speed v = v 0 at some inclination with the x-axis. There is a uniform magnetic field B along the x-axis. P strikes a fixed target T on the x-axis for a minimum value of B = B 0 . P will also strike T if

    For c = 2a if, the magnetic field at point P will be zero when

    Consider three long straight parallel wires as shown in figure. Find the force experienced by a 25 cm length of wire C ( μ N)

    Let E and B denote the electric and magnetic fields in a certain region of space. A proton moving with a velocity along a straight line enters the region and is found to pass through it undeflected. Indicate which of the following statements are consistent with the observations:

    Two circular coils A and B having number of turns 3N and N respectively and carrying equal currents in same sense are placed coaxially, such that they subtend the same solid angle at O. Let B 0 be the magnetic field at O due to smaller coil B, then

    Two infinitely long wires are placed perpendicular to the plane of paper. Current in wire A to 4 i 0 outward the plane of paper and current in ‘B’ is i 0 inward the plane of paper. The ∫ −   α +   α B ⋅   d ℓ along the QP is K ( μ 0 i 0 ) . Find the value of K.

    A particle of mass ‘m’ and charge ‘q’ is projected with a velocity v 0 in a viscous medium, where a uniform and constant magnetic field of induction B exists everywhere in a direction perpendicular to the direction of projection of the particle. The force of viscous drag on the particle is given by the law f = − b v , where ‘b’ is a positive constant and v is velocity of the particle. The distance travelled by the particle during a time interval from the instant of projection until velocity vector turns by 2 π radians is m v 0 b ( 1 − e n π b q B ) . Find ‘n’?

    The coils C 1 and C 2 have same number of turns and carry equal currents in the same sense. They subtend the same angle θ at P. If the magnetic field produced by C 1 at P is B, then effective magnetic field that produced due to both C 1     a n d     C 2 will be

    Consider a metallic ring of radius 1m, mass 1kg, and carrying current 1A in a gravity free space in x-y plane with its center at origin as shown. If a uniform magnetic field 3 i ^ + 4 j ^ T is applied, then instantaneous acceleration of point P(which is on the y-axis at the moment) is n π     m / s 2 . Find n.

    Two infinitely long straight current carrying wires are placed parallel to y-axis in xy plane. Each wire carries current I in opposite directions as shown. A charged particle of charge +q and mass m is projected from position P (0, 0, a) with initial velocity v = v j .Then

    In a thin rectangular metallic strip a constant current I flows along the positive x-direction, as shown in the figure. The length, width and thickness of the strip are l, w and d respectively. A uniform magnetic field B is applied on the strip along the positive y-direction. Due to this, the charge carriers experience a net deflection along the z-direction. This results in accumulation of charge carriers on the surface PQRS and appearance of equal and opposite charges on the face opposite to PQRS. A potential difference along the z-direction is thus developed. Charge accumulation continues until the magnetic force is balanced by the electric force. The current is assumed to be uniformly distributed on the cross section of the strip and carried by electrons.

    A charged particle (electron or proton) is introduced at the origin (x=0,y=0,z=0) with a given initial velocity ν . A uniform electric field E and a uniform magnetic field B exist everywhere. The velocity ν , electric field E and magnetic field B are given in columns 1,2 and 3, respectively. The quantities E 0 , B 0 are positive in magnitude. Column 1 Column 2 Column 3 (I) Electron with ν = 2 E 0 B 0 x ^ (i) E = E 0 z ^ (P) B = − B 0 x ^ (II) Electron with ν = E 0 B 0 y ^ (ii) E = − E 0 y ^ (Q) B = B 0 x ^ (III) Proton with ν = 0 (iii) E = − E 0 x ^ (R) B = B 0 y ^ (IV) Proton with ν = 2 E 0 B 0 x ^ (iv) E = E 0 x ^ (S) B = B 0 z ^ In which case will the particle describe a helical path with axis along the positive z direction?

    A charged particle (electron or proton) is introduced at the origin (x=0,y=0,z=0) with a given initial velocity ν . A uniform electric field E and a uniform magnetic field B exist everywhere. The velocity ν , electric field E and magnetic field B are given in columns 1,2 and 3, respectively. The quantities E 0 , B 0 are positive in magnitude. Column 1 Column 2 Column 3 (I) Electron with ν = 2 E 0 B 0 x ^ (i) E = E 0 z ^ (P) B = − B 0 x ^ (II) Electron with ν = E 0 B 0 y ^ (ii) E = − E 0 y ^ (Q) B = B 0 x ^ (III) Proton with ν = 0 (iii) E = − E 0 x ^ (R) B = B 0 y ^ (IV) Proton with ν = 2 E 0 B 0 x ^ (iv) E = E 0 x ^ (S) B = B 0 z ^ In which case will the particle describe a helical path with axis along the positive z direction?

    A long thick solid conductor has a cylindrical cavity of radius 1 m parallel to its axis at a distance of 2m from centre of cylinder. Cylinder have uniform surface current density J = 10 6 π A / m 2 k ^ A charged particle of mass 0.1 kg and charge π coulomb is projected with velocity v at an angle of 60º as shown in figure. For what maximum v, particle is able to strike point B?

    A particle of specific charge π C/kg , is projected with a speed u from origin along positive direction of x-axis. There exists a magnetic field B = 1 ( − k ^ ) Tesla in the region. Choose the correct option(s).

    Two infinitely long wires are placed perpendicular to the plane of paper. Current in wire A to 4 i 0 outward the plane of paper and current in ‘B’ is i 0 inward the plane of paper. The ∫ −   α +   α B ⋅   d l along the QP is K ( μ 0 i 0 ) . Find the value of K.

    A current flows through a conductor arranged in one plane as shown in figure. The equation of ellipse guiding the diversion of current in the conductor is x 2 a 2 + y 2 b 2 = 1 . Magnetic field due to given current distribution at O is given by μ 0 i n a + m b ( w h e r e   m , n   a r e   n o n − n e g a t i v e ) . Then value of m + n is:

    A conductor of length l and mass m is placed along the east-west line on a horizontal table. Suddenly a certain amount of charge is passed through it and it is found to jump to a height h. The earth’s magnetic field is B (horizontal). The charge passed through the conductor is :

    A current carrying loop is in the shape of an equilateral triangle of side length a. It’s mass is M and it is in vertical plane. There exists a uniform horizontal magnetic field B of magnitude B 0 in the region (up to dotted line) as shown in the diagram. (neglect emf induced in the loop)

    A charge particle A of charge q = 2C has velocity V = 100 m/s. When it passes through point A and has velocity in the direction shown. The strength of instantaneous magnetic field at point B (r = 2m) due to this moving charge is 0.5 × n    μ T . Find n.

    A circular coil of radius R and N turns has negligible resistance. As shown in the schematic figure, its two ends are connected to two wires and it is hanging by those wires with its plane being vertical. The wires are connected to a capacitor with charge Q through a switch. The coil is in a horizontal uniform magnetic field B o parallel to the plane of the coil. When the switch is closed, the capacitor gets discharged through the coil in a very short time. By the time the capacitor is discharged fully, magnitude of the angular momentum gained by the coil will be (assume that the discharge time is so short that the coil has hardly rotated during this time)-

    An electron accelerated through a potential difference V passes through a uniform transverse magnetic field and experiences a force F. If the accelerating potential is increased to 2V, the electron in the same magnetic field will experience a force

    A particle with a specific charge s is fired with a speed v toward a wall at a distance d, perpendicular to the wall.What minimum magnetic field must exist in this region for the particle not to hit the wall?

    Two particles X and Y having equal charges, after being accelerated through the same potential difference, enter a region of uniform magnetic field and describe circular paths of radii R 1 , and R 2 , respectively. The ratio of masses of X and Y is

    An electron of mass 0.90 x 10 -30 kg under the action of a magnetic field moves in a circle of 2.0 cm radius at a speed of 3.0 x 10 6 ms -1 . If a proton of mass 1.8 x 10 -27 kg was to move in a circle of the same radius in the same magnetic field, then its speed will be

    An electron moving with a speed u along the positive x-axis at y = 0 enters a region of uniform magnetic field which exists to the right of y-axis. The electron exits from the region after some time with the speed v at coordinate y, then

    A current carrying loop lies on a smooth horizontal plane.Then,

    If a charged particle of charge to mass ratio ( q / m ) = α enters in a magnetic field of strength B at a speed v = ( 2 α d ) ( B ) , then

    A beam of mixture of α particles and protons are accelerated through same potential difference before entering into the magnetic field of strength B. If r 1 = 5 cm , then r 2 is

    Figure shows an equilateral triangle ABC of side l carrying currents as shown, and placed in a uniform magnetic field B perpendicular to the plane of triangle. The magnitude of magnetic force on the triangle is

    A charged particle moving along +ve x-direction with a velocity v enters a region where there is a uniform magnetic field B 0 ( – k ^ ) , from x = 0 to x = d. The particle gets deflected at an angle θ from its initial path. The specific charge of the particle is

    A proton of mass 1.67 x 10 -27 kg and charge 1.67 x 10 -19 Cs projected with a speed of 2 × 10 6 ms -1 at an angle of 60 0 to the X-axis. If a uniform magnetic field of 0.10 T is applied along Y-axis, the path of proton is

    A particle of charge q and mass m starts moving from the origin under the action of an electric field E = E 0 i ^ and B = B 0 i ^ with a velocity v = v 0 j ^ . The speed of the particle will become 2 v 0 after a time

    A loop of flexible conducting wire of length l lies in magnetic field B which is normal to the plane of loop. A current i is passed through the loop. The tension developed in the wire to open up is

    A charged particle enters a uniform magnetic field with velocity vector at an angle of 45 0 with the magnetic field.The pitch of the helical path followed by the particle is p.The radius of the helix will be

    A uniform magnetic field of 1.5 T exists in a cylindrical region of radius 10.0 cm, its direction being parallel to the axis along east to west. A current carrying wire in north-south direction passes through this region. The wire intersects the axis and experiences a force of 1.2 N downward. If the wire is turned from north south to northeast-southwest direction,then magnitude and direction of force is

    A particle of positive charge q and mass m enters with velocity V j ^ at the origin in a magnetic field B ( – k ^ ) which is present in the whole space. The charge makes a perfectly in elastic collision with an identical particle (having same charge) at rest but free to move at its maximum positive y-coordinate. After collision, the combined charge will move on trajectory (where r = m V q B

    In the plane mirror, the coordinates of image of a charged particle (initially at origin as shown in figure) after two and a half time periods are (initial velocity of charge particle is V 0 , in the x-y plane and the plane mirror is perpendicular to the x-axis. A uniform magnetic field B i ^ exists in the space. P 0 is pitch of helix, R 0 is radius of helix.)

    A particle of specific charge α is projected from origin with velocity v = v 0 i ^ – v 0 k ^ in a uniform magnetic field B = – B 0 k ^ . Find time dependence of velocity of the particle.

    Two straight segments of wire ab and bc each carrying current i, are placed as shown in figure. The cube edge is 50 cm and magnetic field is uniform along Y-axis having magnitude 0.4 T. If i = 3 A, the force experienced by wire abc in the presence of magnetic field is

    The current generator i g , shown in figure, sends a constant current i through the circuit. The wire ab has a length / and mass m , slide on the smooth, horizontal rails connected to i g . The entire system lies in a vertical magnetic field B. The velocity of the wire as a function of time is

    AB and CD are fixed conducting smooth rails placed in a vertical plane and joined by a constant current source at its upper end. PQ is a conducting rod which is free to slide on the rails. A horizontal uniform magnetic field exists in space as shown. If the rod PQ is released from rest then,

    A conducting ring of mass 2 kg and radius 0.5 m is placed‘on a smooth horizontal plane. The ring carries a current of I =4A. A horizontal magnetic field B = 10 T is switched on at time t = 0 as shown in figure. The initial angular acceleration of the ring will be

    A small block of mass m, having charge q, is placed on a frictionless inclined plane making an angle θ with the horizontal. There exists a uniform magnetic field B parallel to the inclined plane but perpendicular to the length of spring. If m is slightly pulled on the incline in downward direction, the time period of oscillation will be (assume that the block does not leave contact with the plane)

    . In figure, there is a uniform conducting structure in which each small square has side a. The structure is kept in a uniform magnetic field B. Then the magnetic force on the structure will be

    In figure, a coil of single turn is wound on a sphere of radius r and mass m. The plane of the coil is parallel to the inclined plane and lies in the equatorial plane of the sphere. If the sphere is in rotational equilibrium, the value of B is[Current in the coil is i]

    Two identical particles having the same mass m and charges+q and -q separated by a distance d enter a uniform magnetic field B directed perpendicular to paper inwards with speeds v 1 and v 2 as shown in figure. The particles will not collide if

    A charged particle of specific charge (charge/mass) o is released from origin at time t= 0 with velocity v = v 0 ( i ^ + j ^ ) in uniform magnetic field B = B 0 i ^ . Coordinates of the particle at time t= π / B 0 α are

    A positive charge particle of mass m and charge q is projected with velocity v as shown in figure. If radius of curvature of charge particle in magnetic field region is greater than d, then find the time spent by the charge particle in magnetic field.

    In a certain region of space, there exists a uniform and constant electric field of strength E along x-axis and uniform constant magnetic field of induction B along z-axis. A charged particle having charge q and mass m is projected with speed v parallel to x-axis from a point (a, b, 0). When the particle reaches a point 2a, b/2, 0 its speed becomes 2v. Find the value of electric field strength in terms of m, v and co-ordinates.

    A particle of specific charge q / m = π C / kg is projected from the origin towards positive x-axis with a velocity of 10 m/sin a uniform magnetic field B = – 2 k ^ T e s l a . The velocity V of the particle after time t= 1/6 s will be

    The torque experienced by a given current carrying loop in a uniform magnetic field B given by B 0 ( i – j ) would have magnitude

    A charged particle (charge q, mass m) has velocity v 0 , at origin in +x direction. In space there is a uniform magnetic field B in -z direction. Find the y coordinate of particle when it crosses y axis.

    Two rectangular plates A and B placed at a distance 2a apart, are connected to a battery to produce an electric field. There are insulators between plates C and other two plates. A magnetic field exists along z-axis. A charged particle of mass m and charge q passes through a hole at the middle of the plate A with velocity v and strikes at Q which is the middle of the bottom edge of plate B after passing through a hole in plate C. If E=m v^{2} / q a what will be the speed of the particle at Q?

    A magnetic field B exists between OA and OB. Inclined at an angle q, a charged particle strikes at point A on surface OA,at a distance ( 2 K cos θ ) / ( q v B ) from O, where K is kinetic energy, q is the charge and v is the velocity of the particle.At what angle with horizontal (measured from end B) will the charged particle emerge from OB?

    To the right of line PQ is a uniform magnetic field B. B 1 A is the line of incidence of a charged particle, which comes out of the field along DE; CA and DF are the normals at A and D. B 1 AC = θ . Angle measured from CA in clockwise direction is taken as positive. The maximum range of movement of the centre of the part of the circle from line AD in which charged particle of charge Q moves with a velocity v when θ is positive to when θ is negative is given by

    To the right of line PQ is a uniform magnetic field B. B 1 A is the line of incidence of a charged particle, which comes out of the field along DE; CA and DF are the normals at A and D. B 1 AC = θ . Angle measured from CA in clockwise direction is taken as positive. The maximum range of movement of the centre of the part of the circle from line AD in which charged particle of charge Q moves with a velocity v when θ is positive to when θ is negative is given by

    To the right of line PQ is a uniform magnetic field B. B 1 A is the line of incidence of a charged particle, which comes out of the field along DE; CA and DF are the normals at A and D. B 1 AC = θ . Angle measured from CA in clockwise direction is taken as positive. The maximum range of movement of the centre of the part of the circle from line AD in which charged particle of charge Q moves with a velocity v when θ is positive to when θ is negative is given by

    A triangular system of mass 100 g consisting of 3 wires of lengths, as shown in figure and length of AO as 4 units, are placed in the magnetic field of 1T. The current of 1A flows through wire AO. The wires are of same material and cross-sectional area. what will be acceleration of the system (in m/sec 2 ) ?

    A triangular system of mass 100 g consisting of 3 wires of lengths, as shown in figure and length of AO as 4 units, are placed in the magnetic field of 1T. The current of 1A flows through wire AO. The wires are of same material and cross-sectional area. what will be acceleration of the system (in m/sec 2 ) ?

    Find the force acting on rectangular loop PORST.

    A charged particle moves insides a pipe which is bent as mv shown in figure. If R < mv qB , then force exerted by the pipe on charged particle at P is (Neglect gravity)

    In figure. a coil of single turn is wound on a sphere of radius r and mass m. The plane of the coil is parallel to the inclined plane and lies in the equatorial plane of the sphere. If the sphere is in rotational equilibrium, the value of B is [Current in the coil is i]

    To the right of line PQ is a uniform magnetic field B . B 1 A is the line of incidence of a charged particle, which comes out of the field along DE; CA and DF are the normals at A and D. B 1 AC = θ . Angle e measured from CA in clockwise direction is taken as positive. What will be the value of θ so that angle subtended by the part that angle subtended by the part of the circle (along which charged particle moves in the field) at its centre and facing the circle is less than π ?

    A charged particle is fired at an angle θ to a uniform magnetic field directed along the x-axis. During its motion along a helical path, the particle will

    A particle having a mass of 0.5 g carries a charge of 2.5 × 10 − 8 C . The particle is given an initial horizontal velocity of 6 × 10 4 ms − 1 . To keep the particle moving in a horizontal direction

    Chat on WhatsApp Call Infinity Learn

      Talk to our academic expert!



      +91


      Live ClassesBooksTest SeriesSelf Learning




      Verify OTP Code (required)

      I agree to the terms and conditions and privacy policy.